AUD Becker Mock Exam 1

¡Supera tus tareas y exámenes ahora con Quizwiz!

Cheek & Association, CPAs, render an unmodified opinion on Roy Roger's year-end financial statements, but unbeknownst to the auditors, the financial statements are materially misstated due to a computer error that causes sales revenue for a particular product to be recorded twice. What type of risk is this?

*Audit risk* Audit risk is the risk that the auditor may unknowingly fail to modify appropriately the opinion on financial statements that are materially misstated. In this case, the *auditors did not know that sales were materially overstated* and therefore *failed to appropriately modify the opinion*.

A lack of appropriate controls surrounding Roy Rogers' purchasing function may allow unintentional errors to occur and remain undetected. What type of risk is this?

*Control risk* Control risk is the risk that a material misstatement that could occur in an assertion will not be prevented or detected on a timely basis by the entity's internal control. In this case, Roy Rogers' purchasing function lacks the controls that would prevent or detect errors.

Cheek & Assoc., CPAs, reply primarily on the cash receipts journal and the cash disbursements journal (internally-generated evidence) in its audit of cash. They conclude cash is fairly stated when, in fact, a large withdrawal on December 31 was not recorded, resulting in a material overstatement of the year-end balance. What type of risk is this?

*Detection risk* Detection risk is the risk that the auditor will not detect a material misstatement that exists in an assertion. Detection risk generally relates to the auditor's procedures. In this case, the auditor relied too heavily on internal evidence and therefore did not detect an error in the existence assertion for cash. (Note that a more effective procedure, such as examining the bank statement or obtaining a confirmation from the bank, would have identified this problem.)

Cheek & Assoc., CPAs, chose to perform most of the testing of receivables at interim, and they do not identify a significant error in the recording of receivables that occurred between interim and year-end. What type of risk is this?

*Detection risk* Detection risk is the risk that the auditor will not detect a material misstatement that exists in an assertion. Detection risk generally relates to the auditor's procedures. In this case, the auditors focused their attention on interim testing and did not appropriately address the incremental risk between interim and year-end.

What event would likely cause a *decrease in inventory turnover*?

*Due to an economic downturn, buyers have shifted to economy cars.* Inventory turnover = Cost of goods sold / Average inventory If buyers have shifted to economy cars this means that fewer luxury cars are being sold in Year 2, resulting in a lower cost of goods sold as well as an increase in average inventory. (The journal entry when inventory is sold is a debit to cost of goods sold and a credit to inventory.)

What event would likely cause a *decrease in return on assets*?

*Due to an economic downturn, buyers have shifted to economy cars.* The formula for return on assets is: (Net income / Net sales ) × (Net sales/ Average total assets) OR Net income / Average total assets The ratio decreased in Year 2 as compared to Year 1. This may result from a decrease in net income and/or an increase in average total assets. If buyers have shifted to buying economy cars instead of luxury cars this would result in a decrease in net income as a result of lower sales. Also, average total assets may have increased as a result of higher inventory levels.

Roy Rogers' employees are aware that a large reduction in the workforce is planned. What type of risk is this?

*Fraud risk* Fraud risk is the risk of material misstatement of the financial statements due to fraud. The auditor is required to evaluate fraud risk, and must consider whether and to what extent fraud risk factors (incentives/pressures, opportunity, and rationalization/attitude) exist. *Anticipated future layoffs create a fraud risk* in that they sometimes provide an *incentive for employees to misappropriate assets.*

Top executives at Roy Rogers will receive significant management bonuses if certain financial targets are achieved. What type of risk is this?

*Fraud risk* Fraud risk is the risk of material misstatement of the financial statements due to fraud. The auditor is required to evaluate fraud risk, and must consider whether and to what extent fraud risk factors (incentives/pressures, opportunity, and rationalization/attitude) exist. Management compensation contingent on achieving financial targets creates a fraud risk in that it sometimes provides an incentive for management to intentionally misstate or omit financial statement amounts or disclosures.

What event would likely cause a *decrease in debt to equity*?

*In Year 2, the company made a required principal payment on its long-term debt.* Debt to equity = Total liabilities / Common shareholders' equity The ratio decreased in Year 2 as compared to Year 1. This can result due to a decrease in total liabilities and/or an increase in common shareholders' equity. If the company made a required payment on long-term debt this would decrease total liabilities. Note that an increase in dividends to shareholders or a buyback of stock would decrease shareholders' equity.

What event would likely cause a *decrease in net fixed assets to equity*?

*In Year 2, the company reclassified its international production factories to held for sale.* The formula for net fixed assets to equity is: Net fixed assets / Equity The ratio decreased in Year 2 as compared to Year 1. This can result due to a decrease in net fixed assets and/or an increase in equity. If the company reclassified its international production facilities to held for sale, this would result in a decrease in net fixed assets because long-lived assets classified as held for sale are presented separately from other assets. (Items held for sale get their own individual line item on the balance sheet.)Note that there would be no change in assets when a company removed fully depreciated machinery from its books.

Roy Rogers recently invested, for the first time, in securities for which the accounting is highly complex. What type of risk is this?

*Inherent risk* Inherent risk is the susceptibility of an assertion to material misstatement assuming there are no related controls. Assertions involving *complex calculations* have relatively *high inherent risk.*

Wilson selects a sample of sales orders and, noting that many of the sales orders lack appropriate credit approval, decides to increase substantive testing. As it turns out, Wilson just happened to pick a nonrepresentative sample, and most sales orders in the population at large have in fact been properly approved. What type of risk is this?

*Risk of assessing control risk too high* The risk of assessing control risk too high is the risk that the sample indicates a greater error rate than actually exists in the population at large. In this case, based on the selected sample, Wilson believes that this control is not operating effectively. In fact, appropriate credit approvals are being obtained.

Wilson selects a sample of invoices and, noting that the majority of the invoices are properly approved, reduced the extent of substantive testing. In fact, most of the invoices selected by Wilson happen to have been processed during the first quarter of the year. On April 1, a new accounting manager was hired who did not require the approval of invoices. What type of risk is this?

*Risk of assessing control risk too low* The risk of assessing control risk too low is the risk that the sample indicates a lower error rate than actually exists in the population at large. In this case, Wilson has mistakenly assumed that invoices throughout the year are properly approved, when in fact this control was not operating during the last three quarters of the year. Wilson's sample is not representative of the population and therefore Wilson will make an erroneous decision.

Based on the sample of securities selected, Wilson erroneously concludes dividend income is materially understated. What type of risk is this?

*Risk of incorrect rejection* The risk of incorrect rejection is the risk that a *sample mistakenly indicates a material misstatement* in an account balance *when in fact such balance is fairly stated*. In this example, Wilson *erroneously assumes*, based on the sample, that dividend income is materially understated. In other words, Wilson will mistakenly reject a properly stated account balance.

What event would likely cause an *increase in Days in accounts receivable*?

*Sales managers are recording fraudulent sales to receive larger commissions* Days sales in accounts receivable = Average net receivables / (Net credit sales/365) Fraudulent sales increase receivables, which would result in a larger amount of A/R as fictitious customers that do not pay their A/R balances.

***Which of the following are elements of a firm's quality control that should be considered in establishing its quality control policies and procedures? a. Assigning personnel, client acceptance, and professional development. b. Assigning personnel, client acceptance, and analytical review. c. Assigning personnel, professional development, and analytical review. d. Client acceptance, professional development, and analytical review.

*a. Assigning personnel, client acceptance, and professional development.* Explanation Choice "1" is correct. Assigning personnel to engagements and providing for professional development are part of the human resources element of quality control. Engagement/client acceptance and continuance is another element of quality control. Choices "4", "3", or "2" are incorrect. Analytical review procedures are not an element of a firm's system of quality control.

Things that impair independence

o A threatened (and likely) lawsuit o a credit card balance in > $10,000 o overdue audit fees if overdue by > one year

Statements on Standards for Attestation Engagements (SSAE)

review of Management and Discussion Analysis

An auditor provides positive assurance in all of the following reports *except*: a. A report on compliance with contractual or regulatory requirements related to audited financial statements. b. A report on a specified account in a financial statement. c. A report on other comprehensive basis of accounting financial statements. d. A report on a financial presentation to comply with contractual agreements or regulatory requirements.

*a. A report on compliance with contractual or regulatory requirements related to audited financial statements. * Explanation Choice "1" is correct. *Negative assurance* is provided on *compliance with* contractual or regulatory requirements related to audited financial statements. Choice "2" is incorrect. Positive assurance is provided in a report on a specified account in a financial statement. Choice "3" is incorrect. Positive assurance is provided in a report on other comprehensive basis of accounting financial statements. Choice "4" is incorrect. Positive assurance is provided in a report on a financial presentation to comply with contractual agreements or regulatory requirements.

***A report issued on significant deficiencies in internal control noted during a financial statement audit of a nonissuer should contain all of the following except: a. A statement of compliance with laws and regulations. b. An indication that the purpose of the audit was to report on the financial statements. c. The definition of material weaknesses. d. A restriction on the use of the report.

*a. A statement of compliance with laws and regulations. * Explanation Choice "1" is correct. No statement of compliance with laws and regulations is required in the report. Choices "3", "4", and "2" are incorrect. The definition of material weaknesses, a restriction on the use of the report, and an indication that the purpose of the audit was to report on the financial statements all should be part of the report.

A special report on financial statements prepared on the cash basis of accounting should include: a. A statement that the audit was conducted in accordance with generally accepted auditing standards. b. A disclaimer of opinion, because an auditor should not report on financial statements that are not designed to be in conformity with GAAP. c. A qualified or adverse opinion, due to the departure from GAAP. d. An emphasis-of-matter paragraph including a brief explanation of the cash basis of accounting.

*a. A statement that the audit was conducted in accordance with generally accepted auditing standards.* Explanation Choice "1" is correct. A special report on financial statements prepared on the cash basis of accounting should include, in the auditor's responsibility paragraph, a statement that the audit was conducted in accordance with generally accepted auditing standards. Choice "2" is incorrect. A special report on financial statements prepared on the cash basis of accounting includes positive assurance, not a disclaimer of opinion, regarding whether the financial statements are presented fairly in conformity with the cash basis of accounting. Choice "3" is incorrect. A special report on financial statements prepared on the cash basis of accounting does not require a qualified or adverse opinion. An "unmodified" opinion may be presented stating that the financial statements are presented fairly in conformity with the cash basis of accounting. Choice "4" is incorrect. A special report on financial statements prepared on the cash basis of accounting does include an emphasis-of-matter paragraph, but it does not provide an explanation of the cash basis of accounting. Instead, it refers to the note that discusses the cash basis of accounting.

Which of the following should be included in a management representation letter? a. Acknowledgment of management's responsibility for the design, implementation, and maintenance of internal control. b. The auditor's belief that the effects of any uncorrected misstatements are immaterial to the financial statements taken as a whole. c. The auditor's belief that the financial statements are fairly presented in conformity with GAAP. d. A disclaimer indicating that management is not responsible for unrecorded transactions of which it is unaware.

*a. Acknowledgment of management's responsibility for the design, implementation, and maintenance of internal control.* Explanation Choice "1" is correct. Specific representations should be obtained regarding management's responsibility for the design, implementation, and maintenance of internal control. Choice "2" is incorrect. Specific representations should be obtained regarding management's (and not the auditor's) belief that the effects of any uncorrected misstatements are immaterial to the financial statements taken as a whole. Remember that the letter is coming from management to the auditor, and so should reflect management's beliefs. Choice "3" is incorrect. Specific representations should be obtained regarding management's (and not the auditor's) belief that the financial statements are fairly presented in conformity with GAAP. Remember that the letter is coming from management to the auditor, and so should reflect management's beliefs. Choice "4" is incorrect. Specific representations should be obtained regarding the absence of unrecorded transactions. A disclaimer stating that management is not responsible for unrecorded transactions would not be acceptable.

Which of the following best describes a difference between an audit and a review of a nonpublic entity's annual financial statements? a. An audit requires an understanding of internal control, whereas a review does not. b. A review requires inquiry and analytical review procedures, whereas an audit requires only more corroborative auditing procedures. c. A review does not require the accountant to be independent, whereas an audit does require independence. d. An audit provides assurance regarding the financial statements, whereas a review provides no assurance.

*a. An audit requires an understanding of internal control, whereas a review does not.* Explanation Choice "1" is correct. Auditing standards require that, during the planning stage, the accountant obtain a sufficient understanding of the entity and its environment, including its internal control. A review of annual financial statements does not contemplate obtaining an understanding of internal control. Note: An interim review (where the annual financial statements are audited) falls under SAS or PCAOB requirements for a nonissuer and issuer, respectively, and would require an understanding of internal control. Choice "2" is incorrect. Although it is true that a review requires inquiry and analytical review procedures, this is not the best answer as to the difference between a review and an audit. Inquiry and analytical review procedures are also required as part of an audit engagement. Choice "3" is incorrect. Both an audit and a review require independence. Choice "4" is incorrect. A review provides negative assurance regarding the financial statements, so it is not correct to say that a review provides no assurance. Both audits and reviews provide some level of assurance regarding the financial statements.

***The purpose of segregating the duties of hiring personnel and distributing payroll checks is to separate the: a. Authorization of transactions from the custody of related assets. b. Human resources functions from the accounting functions. c. Administrative function from the hiring function. d. Application controls from the general controls.

*a. Authorization of transactions from the custody of related assets.* Explanation Choice "1" is correct. The hiring function provides authorization for payment. Distributing payroll is a custodial function. Choice "2" is incorrect. Segregation of duties generally refers to the separation of authorization, recordkeeping and custody, not the separation of various functional units. Choice "3" is incorrect. Hiring is an administrative function. Choice "4" is incorrect. Hiring and payroll distribution are not types of application controls or general controls.

Information gathered in the course of an independent audit is the property of the auditor. This information is not generally disclosed to outside parties. However, after discussion with legal counsel, the auditor may wish to disclose information about irregularities or noncompliance with laws and regulations to outside parties in which of the following circumstances? a. Change of auditor, predecessor/successor communications, and subpoena. b. Predecessor/successor communications, subpoena, and accounting changes. c. Change of auditor, subpoena, and accounting changes. d. Change of auditor, predecessor/successor communications, and accounting changes.

*a. Change of auditor, predecessor/successor communications, and subpoena.* Explanation Choice "1" is correct. A duty to disclose such information outside the entity may exist when there is a change of auditor (reported to the SEC on Form 8-K), in response to a subpoena, and (with client permission) in response to a successor auditor's inquiries. Choices "4", "2", or "3" are incorrect. Accounting changes do not require disclosure to outside parties.

The primary reason an auditor requests that letters of inquiry be sent to a client's attorney is to provide the auditor with: a. Corroboration of information furnished by management about litigation, claims, and assessments. b. An estimate of the amount or range of potential loss. c. An evaluation of the likelihood of an unfavorable outcome. d. A description and evaluation of litigation, claims, and assessments that existed at the balance sheet date.

*a. Corroboration of information furnished by management about litigation, claims, and assessments. * Explanation Choice "1" is correct. It is management's responsibility to identify and account for litigation, claims, and assessments. The letter of audit inquiry serves to corroborate information provided by management. Choices "3" or "2" are incorrect. It is management's responsibility to determine the expected outcome and range of potential loss. While the attorney does provide a professional opinion regarding these estimates, the primary purpose of the attorney's letter is to corroborate the information provided by management. Choice "4" is incorrect. It is management's responsibility to describe and evaluate litigation, claims, and assessments. While the client's attorney should provide a professional opinion regarding such matters, the primary purpose of the attorney's letter is to corroborate management representations.

Which of the following procedures are required for a compilation engagement? I. Read the financial statements. II. Obtain an understanding of the client's business and the accounting principles used. III. Assess fraud risk. a. I and II, only. b. II and III, only. c. I and III, only. d. I, II, and III.

*a. I and II, only.* Explanation Choice "1" is correct. In a compilation engagement, the accountant is required to read the financial statements and to have a general understanding of the client's business and of the accounting principles and practices used by the client. There is *no requirement that the accountant specifically assess fraud*, although a *discovery of fraud cannot be ignored.*

Which of the following most likely would be an internal control procedure designed to detect errors and irregularities concerning the custody of inventory? a. Independent comparisons of finished goods records with counts of goods on hand. b. Approval of inventory journal entries by the storekeeper. c. Segregation of functions between general accounting and cost accounting. d. Periodic reconciliation of work in process with job cost sheets.

*a. Independent comparisons of finished goods records with counts of goods on hand.* Explanation Choice "1" is correct. Independently comparing inventory records with physical inventory counts may detect discrepancies concerning the custody of inventory. Choice "2" is incorrect. Requiring approval of journal entries relates to the recordkeeping aspects of inventory and not to its physical custody. Choice "3" is incorrect. Segregation of duties between general accounting and cost accounting relates to the recordkeeping aspects of inventory and not to its physical custody. Choice "4" is incorrect. Periodic reconciliation of work in process with job cost sheets relates to the recordkeeping aspects of inventory and not to its physical custody.

Which of the following procedures would an auditor be least likely to use in an effort to obtain evidence regarding subsequent events? a. Investigating personnel changes which occurred after year-end. b. Making inquiry of management about unusual adjustments after year-end. c. Obtaining lists of litigation for review with the client's attorneys. d. Reading minutes of board meetings and reading interim financial statements.

*a. Investigating personnel changes which occurred after year-end.* Explanation Choice "1" is correct. Personnel changes generally would not have financial statement implications. Choice "2" is incorrect. The auditor should inquire of management regarding any material unusual adjustments made after year-end, because such adjustments might require financial statement adjustment or disclosure. Choice "3" is incorrect. The auditor should inquire of the client's legal counsel concerning litigation, claims, and assessments, because such issues might have financial statement implications. Choice "4" is incorrect. The auditor should read the minutes of board meetings and examine the latest available interim financial statements to determine whether there are any items that might have financial statement implications.

Which of the following is true? a. An integrated audit is not required but is recommended for both issuers and nonissuers. b. An integrated audit is required for both issuers and nonissuers. c. An integrated audit may be performed for a nonissuer, but it is not required. d. An integrated audit may be performed for an issuer, but it is not required.

*c. An integrated audit may be performed for a nonissuer, but it is not required.* Explanation Choice "3" is correct. If the client is an issuer, then the auditor must perform an integrated audit of the client's financial statements and internal controls over financial reporting. Integrated audits may also be performed for nonissuers, but are not required.

***Plinth Industries, an issuer, employs several former employees of its auditor, Lawrence & Whitney, CPAs. Adams, CEO, began his career with L&W, but left ten years ago; Brooks, CFO, was with L&W until 18 months ago, and James, internal auditor, left L&W six months prior to the start of the current year's audit. Which of the following properly describes the effect of these potential conflicts of interest? a. L&W may audit Plinth. b. L&W may not audit Plinth due to the employment of Brooks within the two-year period preceding the audit. c. L&W may not audit Plinth because the CEO and CFO are both former L&W employees. d. L&W may not audit Plinth due to the employment of James within the one-year period preceding the audit.

*a. L&W may audit Plinth. * Explanation Choice "1" is correct. L&W may audit Plinth since none of the situations described presents a conflict of interest as described in SOX Title II. Choices "3" and "2" are incorrect. Neither Adams nor Brooks was employed by L&W within the one-year period preceding the audit, so neither position poses a conflict of interest as described in SOX Title II. Choice "4" is incorrect. The conflict of interest provisions of SOX Title II apply only to the issuer's CEO, CFO, Controller, or Chief Accounting Officer (or to any person serving in an equivalent position), not to an internal auditor

***An auditor uses variables sampling techniques to project the inventory balance each year for comparison with the client's assertion regarding the inventory balance in the financial statements. In 20X7, the auditor's sample size was 112 inventory items. In 20X8, the number of units in inventory, the tolerable misstatement, and the required confidence were the same as in 20X7; however, the population variability increased due to the introduction of a new product line. The 20X8 required sample size would, therefore, be: a. Larger than the 20X7 required sample size. b. Smaller than the 20X7 required sample size. c. The same as the required sample size in 20X7. d. Indeterminate based on the information provided.

*a. Larger than the 20X7 required sample size.* Explanation Choice "1" is correct. Population variability has direct effect on sample size. Choices "2", "3", or "4" are incorrect, as population variability has direct effect on sample size.

Which of the following properly describes the *auditor's responsibilities* as opposed to *management's responsibilities*? a. Management is responsible for affirming that the effects of any uncorrected misstatements in the financial statements are immaterial and the auditor is responsible for obtaining reasonable assurance about whether the financial statements are free of material misstatement. b. The auditor is responsible for the entity's financial statements and management is responsible for the selection and application of accounting principles. c. Management is responsible for the entity's financial statements and the auditor is responsible for the selection and application of accounting principles. d. The auditor is responsible for identifying the laws and regulations applicable to the entity's activities and management is responsible for affirming that the effects of any uncorrected misstatements in the financial statements are immaterial.

*a. Management is responsible for affirming that the effects of any uncorrected misstatements in the financial statements are immaterial and the auditor is responsible for obtaining reasonable assurance about whether the financial statements are free of material misstatement.* Explanation Choice "1" is correct. Management is responsible for affirming that the effects of any uncorrected misstatements in the financial statements are immaterial and the auditor is responsible for obtaining reasonable assurance about whether the financial statements are free of material misstatement. Choices "2" and "3" are incorrect. Management is responsible for both the entity's financial statements and for the selection and application of accounting principles. Choice "4" is incorrect. Management is responsible for both identifying the laws and regulations applicable to the entity's activities, and for affirming that the effects of any uncorrected misstatements in the financial statements are immaterial.

As the acceptable level of detection risk decreases, an auditor may change the: a. Nature of substantive tests from a less effective to a more effective procedure. b. Timing of substantive tests from year-end to an interim date. c. Assessed level of control risk from high to low. d. Assessed level of inherent risk.

*a. Nature of substantive tests from a less effective to a more effective procedure.* Risk assessment based on the operation of IC = *DR increases = Substantive tests decrease* Explanation Choice "1" is correct. *Better evidence must be obtained to achieve a lower level of detection risk.* (*better evidence = lower risk*) Choice "2" is incorrect. Shifting tests from year-end to interim increases detection risk. Choice "3" is incorrect. The assessed level of control risk (based on the auditor's evaluation of the client's controls) determines the risk of material misstatement, which in turn affects the acceptable level of detection risk. The acceptable level of detection risk is therefore determined by the assessed level of control risk, not vice versa. Choice "4" is incorrect. The assessed level of inherent risk (based on the auditor's evaluation of the nature of the assertion) determines the risk of material misstatement, which in turn affects the acceptable level of detection risk. The acceptable level of detection risk is therefore determined by the assessed level of inherent risk, not vice versa.

Which of the following ratios would most likely be used to evaluate an entity's profitability? a. Net income / net sales b. Dividends per common share / earnings per share c. Net sales / average total assets d. Cost of goods sold / average inventory

*a. Net income / net sales* Explanation Choice "1" is correct. Net income divided by net sales, which equals net profit margin, indicates the percentage of every sales dollar that becomes profit. Choice "2" is incorrect. Dividends per common share divided by earnings per share measures the percentage of earnings being paid to shareholders, but it does not indicate the company's overall profitability. Choice "3" is incorrect. Net sales divided by average total assets measures how effectively assets are used. Choice "4" is incorrect. Cost of goods sold divided by average inventory measures how quickly inventory is sold.

*** The auditor concluded that the disclosures made in the financial statements *did not adequately inform* financial statement users about the company's ability to continue as a going concern. Under these circumstances, the auditor should issue a(n): a. Qualified (except for) or adverse opinion, depending on the level of materiality. b. Qualified (except for) or disclaimer of opinion depending on the level of materiality. c. Unmodified opinion with an emphasis-of-matter paragraph or disclaimer of opinion, depending on the level of materiality. d. Adverse opinion or a disclaimer of opinion.

*a. Qualified (except for) or adverse opinion, depending on the level of materiality.* Explanation Choice "1" is correct. Improper disclosure of a going concern situation is a GAAP violation, so a qualified or adverse opinion should be issued. Choices "2" and "4" are incorrect. While the auditor is never precluded from issuing a disclaimer of opinion, the direct result of inadequate disclosure is either a qualified or adverse opinion. Choice "3" is incorrect. The opinion *would be unmodified* with an emphasis-of-matter paragraph when there is a going concern issue that has been *adequately disclosed*. The auditor is never precluded from issuing a disclaimer of opinion, but this is not the general result of inadequate disclosure.

An auditor concerned with the completeness of dividend income would most likely: a. Review dividend record books produced by outside service companies. b. Trace recorded dividend receipts to inclusion in the cash receipts journal. c. Review the minutes of board of directors meetings for approval of dividends. d. Send confirmations to a sample of companies in which the client owns stock.

*a. Review dividend record books produced by outside service companies.* Explanation Choice "1" is correct. The auditor would likely choose a sample of companies in which the client owns stock, and review dividend record books (such as Moody's) to determine whether such companies declared dividends during the year under audit. Choice "2" is incorrect. An auditor concerned with the completeness assertion would be searching for unrecorded dividends. Starting an audit test with recorded dividend receipts would not provide any evidence about those items which may have been unrecorded. Choice "3" is incorrect. The board of directors does not approve dividends coming from other companies, only dividends being declared by their own company. Choice "4" is incorrect. Confirmations are not generally used to audit dividend income, nor would this be particularly efficient. Reference to dividend record books produced by investment advisory services such as Moody's is a much faster way to audit dividend income.

When assessing the competence of an entity's internal auditor, an independent CPA should obtain information about all of the following except: a. The organizational status of the internal auditor. b. Quality of the internal auditor's audit documentation. c. Professional certification of the internal auditor. d. Education level and professional experience of the internal auditor.

*a. The organizational status of the internal auditor.* Explanation Choice "1" is correct. The organizational status of the internal auditor is used to evaluate objectivity. Choice "2" is incorrect. An internal auditor's competence may be evaluated based upon the quality of his or her audit documentation. Choice "3" is incorrect. An internal auditor's competence may be evaluated based upon his or her professional certification. Choice "4" is incorrect. An internal auditor's competence may be evaluated based upon his or her educational level and professional experience

Which of the following is *least* likely to be used as a substantive test relating to cash balances? a. Verify that cash disbursements have been properly approved. b. Send cash confirmations to all banks with whom the client has done business. c. Count all cash on hand. d. Obtain cutoff bank statements and perform bank reconciliations.

*a. Verify that cash disbursements have been properly approved.* Explanation Choice "1" is correct. Verifying that cash disbursements have been properly approved is a test of controls, not a substantive test. Choices "2", "3", and "4" are incorrect. Cash confirmations, cash counts, and bank reconciliations are all a means of verifying the ending cash balance.

Kent Industries anticipates that its sales revenue will grow by 5% during the coming year based on historical returns, but also believes there is a slight chance for 20% growth if a new product that is currently being developed is launched during the year. Kent's management has prepared two sets of financial statements, one based on 5% growth and the other based on 10% growth, in an effort to plan for the future. Giacomo, CPA, has been hired to examine both sets of financial statements. Which of the following is true regarding Giacomo's two reports? a. The 5% growth financial statements would include a restriction on the use of the report, whereas the 10% growth financial statements would not include such a restriction. b. Both the report on the 10% growth financial statements and the report on the 5% growth financial statements are appropriate for limited use. c. The report on the 10% growth financial statements would include a statement that the results might not be achieved, whereas the 5% growth financial statements would not require such a comment. d. Neither the report on the 10% growth financial statements nor the report on the 5% growth financial statements should provide positive assurance.

*b. Both the report on the 10% growth financial statements and the report on the 5% growth financial statements are appropriate for limited use. * *Note* that the *5% growth financial statements are a financial forecast* (based on *expected results*) whereas the *10% growth financial statements are a financial projection* (based on the *hypothetical release* of the new product). Choice "2" is correct. Both financial forecasts and financial projections are appropriate for limited use. Choice "1" is incorrect. Use of the report on the 10% growth financial statements (projection) would be restricted; use of the report on the 5% growth financial statements (forecast) could be restricted or unrestricted. Choice "3" is incorrect. Both financial forecasts and financial projections should include a statement that the results might not be achieved. Choice "4" is incorrect. Examinations of prospective financial statements include positive assurance regarding whether the statements are presented in conformity with AICPA guidelines, and whether the underlying assumptions provide a reasonable basis for the financial statements. Both reports would therefore include positive assurance.

***The accountant may report on agreed-upon procedures applied to specified elements, accounts, or items of financial statements: a. But must be independent of the client company and provide limited assurance on the sufficiency of the agreed-upon procedures performed. b. But must be independent of the client company and give no opinion or any other form of assurance on the sufficiency of the agreed-upon procedures applied. c. As long as the accountant assumes responsibility for the adequacy of the agreed-upon procedures the accountant has agreed to perform. d. Without being independent of the client company since no opinion or any other form of assurance is given on the sufficiency of agreed-upon procedures performed.

*b. But must be independent of the client company and give no opinion or any other form of assurance on the sufficiency of the agreed-upon procedures applied.* Explanation Choice "2" is correct. The accountant must be independent to perform an agreed-upon procedures engagement, and should not provide an opinion or any other form of assurance on the sufficiency of the procedures applied. The specified party (often the client) is responsible for the sufficiency of the procedures. Choice "1" is incorrect. The accountant should not provide an opinion or any other form of assurance on the sufficiency of the procedures applied. The specified party (often the client) is responsible for the sufficiency of the procedures. Choice "3" is incorrect. The specified party (often the client) is responsible for the adequacy of the procedures. Choice "4" is incorrect. The accountant must be independent to perform an agreed-upon procedures engagement

Which of the following impairs independence? I. A threatened (and likely) lawsuit related to audit deficiencies. II. Audit fees that are 120 days overdue. III. A $15,000 credit card balance from a financial institution client. a. II and III, only. b. I and III, only. c. I, II, and III. d. I and II, only.

*b. I and III, only.* Explanation Choice "2" is correct. A threatened (and likely) lawsuit and a credit card balance in excess of $10,000 impair independence, but *overdue audit fees only impair independence if they are overdue by more than one year.*

Confidential client information may be disclosed by the accountant without the client's permission in all of the following circumstances except: a. As part of a quality review of the member's professional practices. b. In response to an inquiry by a successor auditor. c. In response to an inquiry made by the ethics division of the AICPA. d. If necessary to comply with a validly issued subpoena.

*b. In response to an inquiry by a successor auditor.* Explanation Choice "2" is correct. Confidential client information may not be disclosed to a successor auditor without the client's permission. Choice "1" is incorrect. Confidential client information may be disclosed by the accountant as part of a quality review of the member's professional practices. Choice "3" is incorrect. Confidential client information may be disclosed by the accountant in response to an inquiry made by the ethics division of the AICPA. Choice "4" is incorrect. Confidential client information may be disclosed by the accountant if necessary to comply with a validly issued subpoena.

Whitney, CPA, is assessing the auditability of Nissen Manufacturing, a possible new client. Which of the following would be least likely to cause Whitney to reject Nissen as a new client? a. Management of Nissen expresses a disregard for maintaining an adequate internal control environment. b. Management of Nissen is unwilling to send accounts receivable confirmations, due to a desire not to trouble customers. c. Management of Nissen is unwilling to provide all financial records to Whitney, due to a desire to keep such information confidential. d. Management of Nissen is unable to provide financial records for the second half of the year due to a computer malfunction.

*b. Management of Nissen is unwilling to send accounts receivable confirmations, due to a desire not to trouble customers.* Explanation Choice "2" is correct. If a potential client is unwilling to send accounts receivable confirmations, the auditor may perform alternative procedures, such as reviewing subsequent cash receipts. Choice "1" is incorrect. Management's disregard for its responsibility to maintain an adequate internal control environment compromises its ability to provide reasonable assurance regarding reliable financial reporting and may lead the auditor to decide not to accept a new engagement because the risk of financial statement misstatement is too high. Choices "4" and "3" are incorrect. If a potential client is unable or unwilling to provide the financial information needed by the auditor to complete the audit, the auditor may not be willing to accept the engagement.

Min, CPA, is *unable to perform necessary procedures* in a review of the financial statements of a nonissuer. What will be the effect on Min's review report? a. Min should issue a review report with an additional paragraph inserted before the final paragraph describing the situation. b. Min should not issue a review report because the review is incomplete. c. Min should issue a review report with an additional paragraph inserted before the final paragraph describing the situation, and a final paragraph that includes the words "except for." d. Min should not issue a review report unless the scope restriction was caused by circumstance, rather than by action of the client.

*b. Min should not issue a review report because the review is incomplete. * Explanation Choice "2" is correct. Accountants *must be able to perform whatever procedures they deem necessary*, and if those procedures are *not* accomplished, the *review is incomplete*. A review that is incomplete will *prevent the issuance of a review report.* Choice "1" is incorrect. Adding an additional paragraph is not an appropriate response. If the review is incomplete, no review report should be issued. Choice "3" is incorrect. Adding an additional paragraph is not an appropriate response. If the review is incomplete, no review report should be issued. Choice "4" is incorrect. Regardless of the reason for the scope restriction, a review that is incomplete will prevent the issuance of a review report.

***Which of the following services require the accountant to comply with Statements on Standards for Accounting and Review Services? a. Maintaining depreciation schedules. b. Preparing financial statements prior to audit or review by another accountant. c. Preparing financial statements in conjunction with litigation services that involve pending regulatory proceedings. d. Processing payments in an accounting software system.

*b. Preparing financial statements prior to audit or review by another accountant. * Explanation Choice "2" is correct. Preparation of financial statements prior to audit or review by another accountant is an engagement that should be conducted in accordance with SSARS. Choice "3" is incorrect. SSARS explicity excludes services involving preparing financial statements in conjunction with litigation services that involve regulatory proceedings. Choices "4" and "1" are incorrect. Processing payments and maintaining depreciation schedules are bookkeeping services, and, therefore, do not require complying with SSARS.

***In testing controls over cash disbursements, an auditor most likely would determine that the person who signs the check also: a. Is denied access to the supporting documents. b. Stamps, perforates or otherwise cancels supporting documents. c. Returns the checks to accounts payable. d. Approves the voucher for payment.

*b. Stamps, perforates or otherwise cancels supporting documents.* Explanation Choice "2" is correct. This prevents duplicate payments of vendor invoices since cancellation occurs immediately after the check is signed. Choice "1" is incorrect. The person who signs the check should have access to the supporting documents, as they must be cancelled after payment. Choice "3" is incorrect. Accounts payable should not get the signed checks, since this would allow one group to have both recordkeeping and custodial duties. The check signer should mail the check. Choice "4" is incorrect. The responsibility for authorization and custody should be separated.

Which of the following standards should a CPA firm apply in a review of Management Discussion and Analysis? a. Statements on Standards for Accounting and Review Services. b. Statements on Standards for Attestation Engagements. c. Statements on Auditing Standards. d. Statements on Standards for Consulting Services.

*b. Statements on Standards for Attestation Engagements.* Explanation Choice "2" is correct. A CPA should refer to Statements on Standards for Attestation Engagements (SSAE) for a review of Management and Discussion Analysis. Choice "1" is incorrect. A CPA should refer to Statements on Standards for Accounting and Review Services when performing a preparation, compilation, or review of historical financial statements for nonissuers. SSARS also applies to preparation and compilations of pro forma and prospective financial information. Choice "3" is incorrect. A CPA should refer to Statements on Auditing Standards when performing an audit for a nonissuer. Choice "4" is incorrect. A CPA should refer to Statements on Standards for Consulting Services when performing consulting services.

Which should be the auditor's primary focus when considering related party transactions? a. The need to disclose payments made to top members of management under compensation arrangements and for expense allowances. b. The adequacy of the client's disclosure of such transactions. c. The determination as to whether such transactions were consummated under arm's-length terms. d. The legality of such transactions.

*b. The adequacy of the client's disclosure of such transactions.* Explanation Choice "2" is correct. The auditor's primary concern with related party transactions is that they are properly disclosed in accordance with GAAP. Choice "1" is incorrect. GAAP requires disclosure of material related party transactions except compensation arrangements, expense allowances, and other similar items in the ordinary course of business. Choice "3" is incorrect. Generally it will not be possible for an auditor to determine whether or not a transaction would have taken place in exactly the same manner if the parties were not related. For this reason, related-party transactions are not considered to be arm's-length transactions. Choice "4" is incorrect. The auditor generally would not be expected to evaluate the legality of related party transactions, although appropriate response would be necessary if the auditor became aware of a possible illegality.

Under which of the following circumstances would an adverse opinion be most appropriate? a. The auditor was unable to observe client inventory counts by virtue of the fact that the auditor was appointed after balance sheet date. b. The financial statements include property, plant, and equipment amounts at fair market value based on management's position that fair market value better depicts true financial position and results of operations of the company. c. There has been a material impact on the financial presentations due to a justifiable change in reporting entity, which has been fully disclosed in the financial statements and related footnotes. d. The client has severely limited the scope of the audit, not permitting access to outside client legal counsel.

*b. The financial statements include property, plant, and equipment amounts at fair market value based on management's position that fair market value better depicts true financial position and results of operations of the company.* Explanation Choice "2" is correct. An adverse opinion is required when serious GAAP problems exist. (GAAP requires that property, plant, and equipment be stated at cost less accumulated depreciation.) Choice "3" is incorrect. A justifiable change in accounting principle does not result in an adverse opinion. Choices "4" and "1" are incorrect. Scope limitations, as opposed to GAAP problems, result in qualified opinions or disclaimer of opinion, not adverse opinions.

***Which risk, when assessed at a high level, is most likely to result in an inappropriate opinion on financial statements which are not fairly stated? a. The risk of incorrect rejection. b. The risk of incorrect acceptance. c. The risk of assessing control risk too low. d. The risk of assessing control risk too high.

*b. The risk of incorrect acceptance.* Explanation Choice "2" is correct. If there is a high risk of incorrect acceptance, this means that it is quite possible that the auditor will incorrectly accept a balance as fairly stated, when in fact it is not fairly stated. Choice "1" is incorrect. If there is a high risk of incorrect rejection, this means that the auditor will do more work than is necessary, which ultimately should lead to a proper opinion on the financial statements. Choice "3" is incorrect. If control risk is assessed too low, the auditor will rely on controls to a greater extent than he/she should. The fact that controls are not functioning as effectively as believed does make it more likely that a misstatement could occur and not be identified, but not quite as likely as is the case with a high risk of incorrect acceptance. Remember that a lack of controls does not imply that there is a misstatement in the financial statements, only that it is more likely. An incorrect acceptance situation does imply that there is an error in the financial statements. Choice "4" is incorrect. If control risk is assessed too high, the auditor will rely less on controls than he/she otherwise might have, and will perform an increased level of substantive testing. The auditor will do more work than is necessary, which ultimately should lead to a proper opinion on the financial statements.

An auditor would express an unmodified opinion with an emphasis-of-matter paragraph added to the report for: A justified change in accounting principle/ An unjustified change in accounting principle/ A justified change in accounting estimate a. No No Yes b. Yes No No c. Yes Yes No d. No Yes Yes

*b. Yes No No * Explanation Choice "2" is correct. *Only a justified change in accounting principle* would result in an unmodified opinion with an emphasis-of-matter paragraph. An *unjustified change* leads to a *qualified or adverse opinion*, and a change in estimate does *not* require an emphasis-of-matter paragraph.

Which of the following are required as part of an auditor's planning process? Understanding the design of controls/ Determining whether controls have been implemented/ Evaluating the operating effectiveness of controls/ Documenting the understanding of internal control a. Yes No No Yes b. Yes Yes No Yes c. No No Yes No d. No Yes Yes No

*b. Yes Yes No Yes* Explanation Choice "2" is correct. As part of planning, the auditor is required to obtain an understanding of the design of controls and determine whether they have been implemented, as well as to document this understanding. The auditor is not required to evaluate the operating effectiveness of controls during the planning process.

***Which of the following is not true about the Department of Labor's independence guidelines? a. The independence guidelines prohibit the accountant from maintaining financial records for the employee benefit plan. b. The independence guidelines apply to rendering an opinion on employee benefit plans. c. An immaterial direct financial interest in the plan does not impair independence. d. A former employee of the plan may be employed by the accounting firm.

*c. An immaterial direct financial interest in the plan does not impair independence.* Explanation Choice "3" is correct. Under the Department of Labor's independence guidelines, any direct financial interest in the plan impairs independence. Choice "1" is incorrect. The Department of Labor's independence guidelines prohibit the accountant from maintaining financial records for the employee benefit plan. Choice "2" is incorrect. The Department of Labor's independence guidelines apply to rendering an opinion on employee benefit plans. Choice "4" is incorrect. Under the Department of Labor's independence guidelines, a former employee of the plan may be employed by the accounting firm provided the individual has completely disassociated from the plan or plan sponsor and the individual does not participate in auditing the financial statements of the plan covering any period of his or her employment by the plan or plan sponsor.

Which of the following statements is true about required procedures in a review of annual financial statements? a. Obtaining an understanding of internal control is not required, but confirmations of accounts receivable is required. b. Obtaining an understanding of the client's operations is not required, but corroboration of management's material estimates is required. c. Assessment of fraud risk is not required, but obtaining a representation letter from management is required. d. Analytical review procedures are not required, but communication with the predecessor accountant is required.

*c. Assessment of fraud risk is not required, but obtaining a representation letter from management is required.* Explanation Choice "3" is correct. Assessment of fraud risk is not required, but obtaining a representation letter from management is required. Choice "1" is incorrect. While it is true that obtaining an understanding of internal control is not required in a review of annual financial statements, confirmation of accounts receivable is an audit procedure that is not required in a review engagement. Note: Obtaining an understanding of internal control is required in a review of interim financial statements (when the annual financial statements are audited). Choice "2" is incorrect. Obtaining an understanding of the client's operations is required, and corroboration of management's material estimates is an audit procedure that is not required in a review engagement. Choice "4" is incorrect. Analytical review procedures are required, whereas communication with the predecessor accountant is not required.

***Which of the following is not true about audit documentation? a. Audit documentation should include identification of the staff who performed the audit work. b. Audit documentation should demonstrate that sufficient appropriate audit evidence has been obtained to support the conclusions reached and the report to be issued. c. Audit documentation should include a separate page for each material account balance or transaction class. d. Audit documentation should include information related to the selection and application of accounting principles.

*c. Audit documentation should include a separate page for each material account balance or transaction class.* Explanation Choice "3" is correct. There is no requirement that documentation related to each material account balance or transaction class be included on a separate page. In fact, it is common for related accounts to be audited and documented together. Choice "1" is incorrect. Audit documentation should include identification of the staff who performed the audit work. Choice "2" is incorrect. Audit documentation should demonstrate that appropriate audit evidence has been obtained to support the conclusions reached and the report to be issued. Choice "4" is incorrect. Audit documentation should include significant audit findings, such as matters that are related to the selection and application of accounting principles.

***How do auditing standards differ from auditing procedures? a. Auditing standards may not apply to all audits, whereas auditing procedures are always necessary to some extent. b. Auditing procedures must be determined first, before auditing standards can be applied. c. Auditing standards apply to all audits, whereas auditing procedures may vary from one engagement to the next. d. Auditing procedures are determined by the AICPA (non-issuers) and the PCAOB (issuers), whereas auditing standards are determined by the auditor based on his/her professional judgment.

*c. Auditing standards apply to all audits, whereas auditing procedures may vary from one engagement to the next.* Explanation Choice "3" is correct. Generally accepted auditing standards apply to all audits, whereas specific audit procedures will vary from one engagement to the next. Choice "1" is incorrect. Auditing standards set a minimum requirement for the profession, and apply to all audits. Choice "2" is incorrect. Auditing standards underly the application of auditing procedures. Audit procedures are developed as a means of achieving auditing standards. Choice "4" is incorrect. Auditing standards are issued by both AICPA and PCAOB, but auditing procedures are determined by the auditor using professional judgment. The auditor determines the nature, extent and timing of audit procedures for each audit.

Which is true about accounts receivable confirmations? a. Confirmations provide evidence about existence, rights, and valuation. b. Blank confirmations provide improved response rates but may be less reliable. c. Confirmations may be based on single transactions rather than entire customer balances. d. The client may mail the confirmations, but they should be returned directly to the auditor.

*c. Confirmations may be based on single transactions rather than entire customer balances.* Explanation Choice "3" is correct. The auditor should consider the types of information respondents will be readily able to confirm. For example, it may be easier for certain respondents to confirm individual transactions rather than entire balances. In such cases, the auditor may choose to confirm based on invoice number (i.e., single transactions). Choice "1" is incorrect. Accounts receivable confirmations do not provide reliable evidence about valuation. *Just because a respondent confirms that he/she owes a certain amount does not mean that he/she has the intent or the ability to pay it.* Choice "2" is incorrect. Blank confirmations are those in which the recipient is requested to fill in the balance. They provide improved reliability since the respondent cannot simply sign off without actually verifying the balance. However, since more work is required, response rates are often decreased. Choice "4" is incorrect. The auditor should maintain control over the confirmation requests and responses, which means the auditor should be the one to mail the confirmations, not the client.

***A report on a nonissuer's internal control based on an audit of internal control should include a statement indicating that: I. Because there are inherent limitations in internal control, misstatements may occur and not be detected. II. Projections of the evaluation of internal control to future periods are subject to the risk that the internal control may become inadequate. III. The report is intended solely for the information and use of the audit committee, management, and other specified parties. a. I and III only. b. I, II, and III. c. I and II only. d. II and III, only.

*c. I and II only.* Explanation Choice "3" is correct. Each of the first two statements would be included in the inherent limitations paragraph of the report. The third statement, indicating restricted use, would not be included as there is no restriction on the use of this report. (A report on internal control matters noted during an audit would, however, include a restricted use paragraph.)

With respect to an auditor's consideration of fraud risk, which of the following is not required? a. A discussion among engagement personnel regarding fraud. b. An assumption that that there is a risk of management override of controls. c. Implementation of controls to detect fraud. d. Incorporation of an element of unpredictability into the audit.

*c. Implementation of controls to detect fraud.* Explanation Choice "3" is correct. It is management's responsibility (not the auditor's) to design and implement programs and controls to prevent, deter, and detect fraud. Choice "1" is incorrect. During planning, engagement personnel are required to discuss the potential for material misstatement due to fraud. Choice "2" is incorrect. There is a presumption in every audit that risk of management override of controls exists. Choice "4" is incorrect. The auditor should incorporate an element of unpredictability into every audit.

***For an entity's financial statements to be presented fairly in accordance with an applicable financial reporting framework, the framework selected should: a. Match the reporting framework used by most other entities within the entity's particular industry. b. Be U.S. GAAP, for all audits performed in the United States. c. Include an adequate description of the framework in the financial statements. d. Be approved by the Auditing Standards Board or the appropriate industry subcommittee.

*c. Include an adequate description of the framework in the financial statements.* Explanation Choice "3" is correct. The preparation and fair presentation of the financial statements requires identification of the applicable financial reporting framework and inclusion of an adequate description of the framework, as well as preparation and fair presentation in accordance with the framework. Choice "1" is incorrect. There is no requirement that an entity's financial statements be prepared in accordance with prevalent industry practices. Choice "2" is incorrect. There may be other financial reporting frameworks, such as IFRS (International Financial Reporting Standards), that are used by companies that are audited in the United States. Choice "4" is incorrect. The Auditing Standards Board does not establish a financial reporting framework.

Which of the following procedures generally would not be performed in a review of a public entity's interim financial statements? a. Read the minutes of stockholder meetings, directors' meetings, etc. b. Compare the current quarter to the comparable quarter from the previous year. c. Inquire of the client's attorney. d. Inquire regarding significant deficiencies in internal control.

*c. Inquire of the client's attorney.* Explanation Choice "3" is correct. Inquiry of the client's attorney generally is not required. Choice "1" is incorrect. The accountant should read the minutes of stockholder meetings, directors' meetings, etc. Choice "2" is incorrect. The accountant should perform analytical procedures, such as comparisons over time, with respect to interim financial information. Choice "4" is incorrect. Inquiry should be made regarding significant deficiencies in internal control.

***Assuming no other material misstatements are found, an independent auditor determines that supplementary information is not fairly stated relative to the basic financial statements taken as a whole. In this instance, the independent auditor should: a. Issue a qualified (except for) or adverse opinion. b. Issue a disclaimer of opinion. c. Issue an unmodified opinion and add an other-matter paragraph to describe the auditor's position on the supplementary information. d. Issue an unmodified opinion without reference to the supplementary information outside the basic financial statements.

*c. Issue an unmodified opinion and add an other-matter paragraph to describe the auditor's position on the supplementary information.* Explanation Choice "3" is correct. Supplementary information is outside the basic financial statements, so problems with this information do not prevent the issuance of an unmodified opinion on the basic financial statements. The situation should, however, be disclosed in an other-matter paragraph to the auditor's report. Choices "1" and "2" are incorrect. Since supplementary information is outside the basic financial statements, an unmodified opinion on the basic financial statements is still appropriate. Choice "4" is incorrect. Auditing standards require the auditor to expand his or her report when required supplementary information is not fairly stated relative to the basic financial statements taken as a whole.

Dante, CPA, is auditing the financial statements of Crest Computing. During the previous year, Kratzke & Kratzke, CPAs, audited Crest's financial statements. Crest has decided to present comparative financial statements for the current year. Which statement is true about Kratzke & Kratzke's report? a. Kratzke & Kratzke should not reissue their report, since they may be unaware of recent circumstances that might have affected the previous year's financial statements. b. Kratzke & Kratzke may reissue their report on the previous statements without performing any additional procedures as long as no changes have been made to those statements. c. Kratzke & Kratzke may reissue their report on the previous statements only after performing limited procedures to evaluate the continuing appropriateness of the report. d. Kratzke & Kratzke should not reissue their report unless Dante agrees to co-sign that report.

*c. Kratzke & Kratzke may reissue their report on the previous statements only after performing limited procedures to evaluate the continuing appropriateness of the report.* Explanation Choice "3" is correct. Kratzke & Kratzke should perform *limited procedures, such as reading the current statements, comparing the current and prior statements, and obtaining representation letters* from Crest's management and from Dante. Choice "1" is incorrect. A predecessor auditor may reissue a previous report after performing certain limited procedures. Choice "2" is incorrect. Certain limited procedures are required to be performed before a predecessor auditor can reissue a previous report. Choice "4" is incorrect. There is no requirement that the successor auditor co-sign the report on the previous year's financial statements, and in fact it would be inappropriate to do so.

SAS (Statements on Auditing Standards)

A CPA should refer to Statements on Auditing Standards when performing an *audit for a nonissuer.*

SSCS (Standards for Consulting Services)

A CPA should refer to Statements on Standards for Consulting Services when performing *consulting services.*

Robbins Company uses an outside service organization called Payroll Plus to process its payroll. Matthews, CPA is the auditor of Payroll Plus, and Stevens, CPA is Robbins' auditor. Stevens' assessment of the risk of material misstatement may be based on the effective operation of controls surrounding payroll if: a. Matthews issues a standard, unmodified opinion on the financial statements of the service organization. b. Matthews' audit report indicates that none of the significant deficiencies in internal control identified at Payroll Plus constitute material weaknesses in internal control. c. Matthews provides a report on controls placed in operation and tests of operating effectiveness. d. Matthews provides a report on controls placed in operation.

*c. Matthews provides a report on controls placed in operation and tests of operating effectiveness.* Explanation Choice "3" is correct. Stevens may assume controls are operating effectively if Matthews provides a report on controls placed in operation and tests of operating effectiveness, and if this report supports a reduction in the assessed level of control risk. Choice "1" is incorrect. An audit report on the financial statements of Payroll Plus does not indicate whether controls are operating effectively. It only indicates that the financial statements are fairly stated, and this could be the case regardless of how controls are functioning. Choice "2" is incorrect. The fact that none of the significant deficiencies in internal control are serious enough to be material weaknesses is not sufficient to support a reduction in the assessed level of control risk. Choice "4" is incorrect. A report on controls placed in operation does not provide evidence about operating effectiveness and therefore cannot be used to support a reduction in the assessed level of control risk.

Auditing standards require that the independent auditor's report shall contain either an expression of opinion regarding the financial statements, taken as a whole, or an assertion to the effect that an opinion cannot be expressed. Which of the following would be prohibited by auditing standards? a. The auditor disclaims an opinion on the entire set of financial statements. b. The auditor disclaims an opinion on the income statement and on the statement of cash flows, but renders an unmodified opinion on the balance sheet. c. None of the other actions are prohibited. d. An auditor is hired to render an opinion on the balance sheet only.

*c. None of the other actions are prohibited.* Explanation Choice "3" is correct. One objective of the auditor in issuing a report is to prevent misinterpretation of the degree of responsibility assumed by the auditor. *Disclaiming an opinion on one or all of the statements is acceptable*, as is a limited reporting engagement in which not all of the financial statements are audited.

A risk assessment based on the effective operation of internal control most likely would involve all of the following, *except*: a. Testing of controls. b. Identifying specific internal control policies and procedures relevant to specific assertions. c. Performing more extensive substantive tests with larger sample sizes than originally planned. d. Identifying the types of potential misstatements.

*c. Performing more extensive substantive tests with larger sample sizes than originally planned.* Explanation Choice "3" is correct. A risk assessment based on the effective operation of internal control increases allowable detection risk, which reduces the required extent of substantive testing. Risk assessment based on the effective operation of IC =*DR increases = Substantive testing decreases* Choice "1" is incorrect. If the auditor's risk assessment is based on the effective operation of internal control, the auditor must test the controls to support that assessment. Choice "2" is incorrect. If the auditor's risk assessment is based on the effective operation of internal control, the auditor must identify and test specific relevant controls to support that assessment. Choice "4" is incorrect. The auditor should always use his or her understanding of internal control to consider the types of potential misstatements that may occur.

Which of the following types of audit evidence is the *least* reliable? a. Bank statement obtained from the client. b. Vendor's invoice. c. Purchase order. d. Canceled check.

*c. Purchase order.* Explanation Choice "3" is correct. A purchase order is *internal* documentation and as such, it is more easily manipulated by the client. Choices "2", "4", or "1" are incorrect. A vendor's invoice, a canceled check, and a bank statement obtained from the client are all considered external evidence, which is less likely to be manipulated than is internal evidence.

Cyrus, CPA is the continuing auditor of Topaz, Inc. During the current year's audit, Cyrus becomes aware of evidence that affects the previous year's statements as well as the opinion that was expressed. Topaz is planning to present comparative financial statements that will include last year's financial statements. How should Cyrus handle this situation? a. Report on both sets of financial statements, using the original opinion on last year's financial statements only after reviewing the previous year's audit documentation to ensure that auditing standards were followed. b. Report only on the current year's financial statements. c. Report on both sets of financial statements, updating the previous opinion for any changes that have occurred. d. Report on both sets of financial statements, using the original opinion on last year's financial statements.

*c. Report on both sets of financial statements, updating the previous opinion for any changes that have occurred.* Explanation Choice "3" is correct. The auditor reports on the financial statements "taken as a whole", which applies to all financial statements presented. Since the auditor's report is generally dated as of the completion of fieldwork for the most recent audit, it is implied that previous reports would be updated. Choice "1" is incorrect. Regardless of the fact that auditing standards may have been followed in the previous year, the auditor still has a responsibility to update the previous report for changes in circumstances. Choice "2" is incorrect. The auditor reports on the financial statements "taken as a whole", which applies to all financial statements presented. Choice "4" is incorrect. Since the auditor's report is generally dated as of the completion of fieldwork for the most recent audit, it is implied that previous reports would be updated. "Update" can mean either to reaffirm the previous opinion, or to change it based on new circumstances.

***Which of the following procedures is an auditor most likely to use to test the *completeness* assertion for fixed assets? a. Comparison of assets acquired during the year to the approved capital budget. b. Comparison of items listed in the fixed asset ledger to actual assets observed by the auditor. c. Review of large items charged to repairs and maintenance. d. Recalculation of depreciation on fixed assets.

*c. Review of large items charged to repairs and maintenance.* Explanation Choice "3" is correct. When the auditor reviews large items charged to repairs and maintenance, he/she is looking for items that may have been erroneously expensed instead of capitalized. The completeness assertion focuses on just that -- items that should have been included in fixed assets but were not. Choice "1" is incorrect. Verifying that acquisitions were properly approved does not address the completeness assertion, since it does not help the auditor identify items that should have been capitalized but were not. Choice "2" is incorrect. Tracing from recorded assets to actual assets tests existence, not completeness. The completeness assertion focuses on assets that are not recorded, so starting with recorded assets will not identify this problem. Choice "4" is incorrect. Recalculation of depreciation addresses the valuation/allocation assertion, not the completeness assertion.

When a client makes extensive use of information technology, the auditor should consider the effect this may have on internal control. Which of the following is least likely to be affected? a. The assessed level of control risk. b. The five components of internal control. c. The audit objectives with respect to evaluating internal control. d. The audit procedures used to evaluate controls.

*c. The audit objectives with respect to evaluating internal control.* Explanation Choice "3" is correct. The client's extensive use of information technology generally would not affect the auditor's objectives, although it might affect how those objectives are achieved. Choice "1" is incorrect. An entity's use of information technology may create additional internal control risks, such as the risk of unauthorized access to data. Choice "2" is incorrect. An entity's use of information technology may affect any of the five components of internal control. Choice "4" is incorrect. An entity's use of information technology will affect the appropriate audit procedures to apply. For example, the extent and complexity of computer operations may require the use of computer-assisted audit techniques.

Which of the following constitutes a *weakness in control* related to the revenue cycle? a. The billing clerk prepares a sales invoice. b. The shipping clerk prepares a bill of lading. c. The cash receipts clerk prepares a credit memo. d. The accounts receivable clerk prepares an aging schedule.

*c. The cash receipts clerk prepares a credit memo.* Explanation Choice "3" is correct. Allowing the cash receipts clerk to prepare a credit memo constitutes an inadequate segregation of duties, because the clerk can misappropriate cash and cover the theft by issuing a credit memo. Choice "1" is incorrect. The billing clerk should prepare a sales invoice. Choice "2" is incorrect. The shipping clerk should prepare a bill of lading. Choice "4" is incorrect. The accounts receivable clerk should prepare an aging schedule.

***In planning the audit, the auditor should consider materiality for the financial statements as a whole in terms of: a. The smallest aggregate level of misstatement that could be considered material to the financial statements taken as a whole. b. The largest aggregate level of misstatement that could be considered material to any one of the financial statements. c. The smallest aggregate level of misstatement that could be considered material to any one of the financial statements. d. The largest aggregate level of misstatement that could be considered material to the financial statements taken as a whole.

*c. The smallest aggregate level of misstatement that could be considered material to any one of the financial statements.* Explanation Choice "3" is correct. Because the financial statements are interrelated, the auditor should consider materiality in terms of the smallest aggregate level of misstatement that could be material to any one of the financial statements. Choice "1" is incorrect. Because the financial statements are interrelated, the auditor should consider materiality in terms of the smallest aggregate level of misstatement that could be material to any one of the financial statements. Choices "4" or "2" are incorrect. The auditor is concerned with identifying the smallest aggregate level of misstatement that would affect the judgment of a reasonable person, not the largest such level. The smallest level provides a cutoff beneath which the auditor need not consider; the largest level provides no such cutoff.

Which of the following is *not* true about those charged with governance of an organization? a. Those charged with governance bear responsibility for overseeing the strategic direction of the organization. b. Those charged with governance bear responsibility for financial reporting. c. Those charged with governance typically include management of the organization. d. Those charged with governance typically include the board of directors.

*c. Those charged with governance typically include management of the organization.* Explanation Choice "3" is correct. Management is not typically included in the term "those charged with governance." Choice "1" is incorrect. The term "those charged with governance" refers to those who bear responsibility to oversee the obligations, financial reporting process, and strategic direction of an entity. Choice "2" is incorrect. The term "those charged with governance" refers to those who bear responsibility to oversee the obligations, financial reporting process, and strategic direction of an entity. Choice "4" is incorrect. The term "those charged with governance" is broadly interpreted to encompass the terms "board of directors" and "audit committee."

***Analytical procedures may be used in various stages of an audit. The use of analytical procedures is required: In the Planning Stage/ As a Substantive Test a. Yes Yes b. No Yes c. Yes No d. No No

*c. Yes No* Explanation Choice "3" is correct. Analytical procedures are required during planning and as a final review. Use of analytical procedures as a substantive test is optional.

***Which of the following is true about comfort letters? a. Comfort letters provide positive assurance regarding unaudited financial statements. b. A comfort letter is a letter from the CPA to management and the board of directors of the company. c. Comfort letters are appropriate for general use. d. A review of interim financial information is required before a comfort letter may be issued.

*d. A review of interim financial information is required before a comfort letter may be issued.* Explanation Choice "4" is correct. A review interim financial information is required before a comfort letter may be issued. Choice "1" is incorrect. Comfort letters provide negative assurance regarding unaudited financial statements. Choice "2" is incorrect. A comfort letter is addressed to the named underwriter, not to management or the board of directors. Choice "3" is incorrect. Comfort letters are generally restricted as to use.

Spring Town is evaluating whether it will need an audit under the Single Audit Act. The town will need to engage an auditor to fulfill the expanded requirements of a single audit if the town: a. Receives more than $750,000 in federal financial assistance. b. Receives more than $300,000 in federal financial assistance. c. Expends more than $300,000 in federal financial assistance. d. Expends more than $750,000 in federal financial assistance.

*d. Expends more than $750,000 in federal financial assistance.* Explanation Choice "4" is correct. Entities that expend more than $750,000 in federal financial assistance are required to receive an audit that complies with the provisions of the Single Audit Act. Choice "1" is incorrect. The criteria used for requiring the Single Audit Act are based on expenditures and not receipts. The expenditure threshold is $750,000. Choice "2" is incorrect. The criteria used for requiring the Single Audit Act are based on expenditures and not receipts. The expenditure threshold is $750,000. Choice "3" is incorrect. Entities that expend more than $750,000 in federal financial assistance are required to receive an audit that complies with the provisions of the Single Audit Act.

Management's written representation to the auditor in connection with a governmental audit would most likely include: a. A statement that management had identified and disclosed all material government programs to the auditor. b. Negative assurance that the government has complied with compliance requirements. c. Representation that all known noncompliance had been reported or negative assurance that other noncompliance likely does not exist. d. A statement that management has disclosed any communications from grantors concerning possible noncompliance.

*d. A statement that management has disclosed any communications from grantors concerning possible noncompliance.* Explanation Choice "4" is correct. The management letter will include a statement that management has disclosed any communications from grantors concerning possible noncompliance. Choice "1" is incorrect. The representation letter should include a statement that management has disclosed all governmental programs to the auditor. Management's representation is not limited to only material government programs. Choice "2" is incorrect. The representation letter should include a statement that management believes that the entity has complied with compliance requirements. *Management representations do not provide negative assurance*. Choice "3" is incorrect. Management will assert that they have disclosed all known noncompliance or positively state that there was no such noncompliance. Management representations do not provide negative assurance.

Which of the following is a required component of the independent auditor's report expressing an unmodified opinion? a. An opinion paragraph including a reference to generally accepted auditing principles. b. An other-matter paragraph including the reason for the unmodified opinion. c. An introductory paragraph including the audit firm name. d. An auditor's responsibility paragraph including a reference to generally accepted auditing standards.

*d. An auditor's responsibility paragraph including a reference to generally accepted auditing standards.* Explanation Choice "4" is correct. An auditor's responsibility paragraph including a reference to generally accepted auditing standards is a required component of the auditor's unmodified opinion. Choice "1" is incorrect. The opinion paragraph includes a reference to generally accepted *accounting* (not auditing) principles. *MUST READ CAREFULLY!!!!!* Choice "2" is incorrect. An other-matter paragraph is not where the reason for an unmodified opinion is expressed. The other-matter paragraph is used when required by GAAS or at the auditor's discretion. It refers to matters other than those presented or disclosed in the financial statements that are relevant to the user's understanding of the audit, the auditor's responsibilities, or the auditor's report. Choice "3" is incorrect. The firm name is generally not included in the introductory paragraph.

A limitation on the scope of an audit sufficient to preclude an unmodified opinion will usually result when the client: a. Refuses to disclose in the notes to the financial statements a significant related party transaction. b. Asks the auditor to report on the balance sheet and not the other basic financial statements. c. Omits the statement of cash flows. d. Does not make the minutes of the Board of Directors meetings available to the auditor.

*d. Does not make the minutes of the Board of Directors meetings available to the auditor.* Explanation Choice "4" is correct. Failure to make the minutes available is a scope limitation sufficient to preclude an unmodified opinion. Choice "2" is incorrect. Reporting on only one financial statement and not the others simply involves a limited reporting objective. The auditor should obtain an understanding of the purpose for which the financial statement is prepared, the intended users, and the steps taken by management to determine that the applicable financial reporting framework is acceptable in the circumstances. As long as these steps are taken by the auditor, auditing a single financial statement would not constitute a limitation on the scope of the audit. Choices "3" and "1" are incorrect. Omission of the statement of cash flows and refusal to disclose a significant related party transaction are examples of GAAP violations, but they do not constitute a limitation on the scope of the audit.

Which of the following ordinarily would *not* be included in the auditor's responsibility regarding accounting estimates? a. Determine whether estimates are properly disclosed in the financial statements. b. Assess management's policies and practices regarding estimates. c. Evaluate whether management's estimates are reasonable. d. Establish a process for preparing accounting estimates.

*d. Establish a process for preparing accounting estimates. * Explanation Choice "4" is correct. It is management's responsibility (and not the auditor's) to establish a process for preparing accounting estimates. Choice "1" is incorrect. The auditor should determine whether estimates are properly disclosed in the financial statements. Choice "2" is incorrect. The auditor should assess management's policies and practices regarding estimates. Choice "3" is incorrect. The auditor should evaluate whether management's estimates are reasonable.

When issuing an unmodified opinion, the auditor who evaluates the audit findings should be satisfied that the: a. Amount of factual misstatements are acknowledged and recorded by the client. b. Amount of identified misstatements are documented in the management representation letter. c. Estimate of the total misstatement includes the adjusting entries already recorded by the client. d. Estimate of the total misstatement is less than a material amount.

*d. Estimate of the total misstatement is less than a material amount.* Explanation Choice "4" is correct. An unmodified opinion states that the financial statements are presented fairly, in all material respects. Accordingly, if the auditor believes that total misstatement (includes factual, judgmental, and projected misstatements) is immaterial, an unmodified opinion is appropriate. Choice "1" is incorrect. Even if the client adjusts the financial statements to correct factual misstatements, the auditor still must feel comfortable that any remaining misstatements would not be material to the financial statements before rendering an unmodified opinion. Choice "2" is incorrect. Documenting misstatements in the management representation letter is not required, nor would it eliminate the need to modify the opinion if the identified misstatements were material. Choice "3" is incorrect. An auditor is not concerned with misstatements already corrected by the client. It is the auditor's estimate of uncorrected--and perhaps unknown--errors that affects the type of opinion rendered.

Which of the following statements regarding control risk is true? a. If control risk is assessed at a low level, an auditor is not required to document the basis for this assessment. b. If control risk is assessed at a high level, an auditor is not required to document the basis for this assessment. c. If control risk is assessed at a low level, substantive testing is likely to be more heavily emphasized in the audit process. d. If control risk is assessed at a high level, substantive testing is likely to be more heavily emphasized in the audit process.

*d. If control risk is assessed at a high level, substantive testing is likely to be more heavily emphasized in the audit process.* Explanation Choice "4" is correct. A high level of control risk results in an increased risk of material misstatement, which should be offset by a reduction in detection risk. This is accomplished by performing more (and/or more effective) substantive tests. Choices "2" or "1" are incorrect. The auditor is always required to document the basis for his or her conclusion about the assessed level of risk. Choice "3" is incorrect. If control risk is assessed at a low level, the auditor is likely to focus more on tests of controls and less on substantive audit procedures.

Which of the following are considered internal control environment factors? a. Detection risk, management philosophy, personnel policies, and accounting systems. b. Detection risk and accounting systems. c. Neither detection risk, management philosophy, personnel policies, nor accounting systems. d. Management philosophy and personnel policies.

*d. Management philosophy and personnel policies.* Explanation Choice "4" is correct. Management's philosophy and human resource policies are both factors included in the control environment. Choices "1" or "2" are incorrect. Accounting systems are part of the information and communication component of internal control, not the control environment. Detection risk is not part of any of the components of internal control. Choice "3" is incorrect. Management's philosophy and human resource policies and practices are both factors included in the control environment.

The auditor performs a preliminary risk assessment for a new client. During the initial fieldwork phase the auditor's test of the client's controls indicate that certain controls are not operating effectively. Under this scenario, the auditor would most likely take which of the following actions? a. Perform additional audit procedures but do not revise the preliminary risk assessment. b. Issue a modified audit report. c. Determine the impact on the client's financial statements without performing additional audit procedures. d. Revise the preliminary risk assessment and modify the planned audit procedures.

*d. Revise the preliminary risk assessment and modify the planned audit procedures.* Explanation Choice "4" is correct. Upon determining that the client's internal controls are not operating effectively, the auditor would then modify the preliminary risk assessment and modify the planned audit procedures. This is part of the auditor's ongoing assessment of risk during an audit. Choice "1" is incorrect. Although the auditor may perform further audit procedures under this scenario, the preliminary risk assessment would first be revised to make that determination. Choice "2" is incorrect. This is too severe of an action and, based on a revised risk assessment and additional audit procedures, may not be warranted. Choice "3" is incorrect. While ultimately the auditor will determine if the control weaknesses will result in a material misstatement of the client's financial statements, the risk assessment should first be revised and the planned audit procedures should be modified.

Which correctly describes the relationship between the PCAOB and the SEC? a. The PCAOB is a subsidiary of the SEC, tasked with registering public accounting firms, establishing rules relating to the preparation of audit reports, and conducting inspections, investigations, and disciplinary proceedings. b. The SEC is subject to the oversight of the PCAOB with respect to conducting inspections, investigations, and disciplinary proceedings. c. The SEC is subject to the oversight of the PCAOB with respect to the registration of public accounting firms that prepare audit reports for issuers. d. The PCAOB is subject to oversight by the SEC, and only accounting firms registered with the PCAOB may prepare audit reports for SEC issuers.

*d. The PCAOB is subject to oversight by the SEC, and only accounting firms registered with the PCAOB may prepare audit reports for SEC issuers.* Explanation Choice "4" is correct. The PCAOB is subject to oversight by the SEC, and only accounting firms registered with the PCAOB may prepare audit reports for SEC issuers. Choice "1" is incorrect. The PCAOB is not a subsidiary of the SEC. It is, however, tasked with registering public accounting firms, establishing rules relating to the preparation of audit reports, and conducting inspections, investigations, and disciplinary proceedings. Choices "3" and "2" are incorrect. The SEC is not subject to the oversight of the PCAOB.

Which of the following statements concerning the auditor's use of the work of a specialist is correct? a. The auditor need not obtain an understanding of the methods and assumptions used by the specialist. b. If the specialist is related to the client, the auditor is not permitted to use the specialist's findings as corroborative evidence. c. The auditor should refer to the work or findings of the specialist in the auditor's report when an unmodified opinion is issued. d. The auditor may engage a specialist and use that specialist's work as audit evidence in performing substantive tests to evaluate material financial statement assertions.

*d. The auditor may engage a specialist and use that specialist's work as audit evidence in performing substantive tests to evaluate material financial statement assertions.* Explanation Choice "4" is correct. The auditor may use the work of a specialist to obtain appropriate audit evidence. Choice "1" is incorrect. The auditor should understand the methods and assumptions used by the specialist. Choice "2" is incorrect. A specialist who is related to the client may be acceptable in some circumstances, but the auditor may choose to perform additional procedures in those cases. The auditor may still use the specialist's findings in such situations. Choice "3" is incorrect. Generally the auditor should not refer to the work or findings of the specialist when an unmodified opinion is issued.

Which of the following inquiries should be made of a predecessor auditor before accepting a new client engagement? a. Specific inquiries of the predecessor regarding audit areas that have required an inordinate amount of time. b. Specific inquiries of the predecessor regarding audit problems that arose from the condition of the accounting system and records. c. Specific inquiries of the predecessor regarding communication to management, the audit committee, and those charged with governance about operational inefficiencies. d. The predecessor's understanding as to the reasons for the change of auditors.

*d. The predecessor's understanding as to the reasons for the change of auditors.* Explanation Choice "4" is correct. The successor auditor is required to make certain inquiries of the predecessor auditor before accepting an engagement, including the predecessor's understanding as to the reasons for the change of auditors. Choices "1" and "2" are incorrect. Specific inquiries of the predecessor regarding matters that the successor believes may affect the conduct of the audit, such as audit areas that have required an inordinate amount of time or audit problems that arose from the condition of the accounting system and records, should be made after acceptance. Choice "3" is incorrect. The auditor is not required to communicate to management, the audit committee, and those charged with governance about operational inefficiencies.

An uncertainty may result in: An unmodified opinion/ A qualified opinion/ An adverse opinion/ A disclaimer of opinion a. Yes No No No b. No Yes Yes Yes c. No Yes No Yes d. Yes Yes Yes Yes

*d. Yes Yes Yes Yes* Explanation Choice "4" is correct. An uncertainty may result in an unmodified opinion if management's analysis is supported and properly recorded or disclosed. An uncertainty for which the auditor is unable to obtain sufficient audit evidence would result in either a qualified opinion or a disclaimer of opinion. If the financial statements are materially misstated due to improper accounting for the uncertainty, a qualified or adverse opinion would result.

SSARS (Statements on Standards for Accounting and Review Services)

-A CPA should refer to Statements on Standards for Accounting and Review Services when performing a *preparation, compilation, or review of historical* financial statements for *nonissuers*. -SSARS also applies to preparation and compilations of pro forma and prospective financial information.

Cosenzo LLP is auditing the Year 1 financial records of Pecoraro LLC, a publicly traded manufacturing company. Pecoraro has a December 31 year-end. Cosenzo performed their year-end audit during January and February of Year 2. The financial statements were issued on March 15, Year 2, the date of audit report. Which of the following subsequent event audit procedure(s) that Cosenzo should perform during the audit? a. Review of post balance transactions b. Perform a search for unrecorded liabilities c. Review client bank reconciliations d. Review management representation letter e. Make inquiries of management regarding any unusual adjustments that may have been made in the month of January and February, Year 2 f. Review of the minutes of a stockholders meeting that took place on January 10, Year 2 g. Perform inventory test counts h. Examine the latest interim financial statements i. Review cash disbursement procedures for proper segregation of duties

a. Review of *post balance transactions* d. Review management *representation letter* e. Make *inquiries of management* regarding any unusual adjustments that may have been made in the month of January and February, Year 2 f. Review of the *minutes* of a stockholders meeting that took place on January 10, Year 2 h. *Examine the latest interim financial statements* *Remember PRIME*: Post BS transactions Representation letter Inquiry Minutes Examine IFS (interim financial statements)


Conjuntos de estudio relacionados

EMT Chapter 6 JBL Vocab/Quiz Questions

View Set

Real Estate Principles 10a Online

View Set

Sociology Chapter 10: Race and Ethnicity

View Set

Distribution & Combine Like Terms

View Set